Bạn chưa đăng nhập. Vui lòng đăng nhập để hỏi bài

Những câu hỏi liên quan
Bùi Sỹ Bình
Xem chi tiết
tran thanh minh
15 tháng 7 2015 lúc 8:09

Làm lại câu a

\(2S=\frac{2}{1.2.3}+\frac{2}{2.3.4}+...+\frac{2}{98.99.100}\)

\(2S=\frac{1}{1.2}+\frac{1}{2.3}+...+\frac{1}{98.99}+\frac{1}{99.100}\)

\(2S=1-\frac{1}{2}+\frac{1}{2}-\frac{1}{3}+...+\frac{1}{98}-\frac{1}{99}+\frac{1}{99}-\frac{1}{100}\)

\(2S=1-\frac{1}{100}\)suy ra \(2S=\frac{99}{100}\)

\(S=\frac{99}{100}:2\)suy ra \(S=\frac{99}{200}\)

tran thanh minh
15 tháng 7 2015 lúc 8:03

a, 2S=\(\frac{2}{1.2.3}+\frac{2}{2.3.4}+...+\frac{2}{98.99.100}\)

\(2S=\frac{1}{1}-\frac{1}{2}+\frac{1}{3}+\frac{1}{2}-\frac{1}{3}+\frac{1}{4}+...+\frac{1}{98}-\frac{1}{99}+\frac{1}{100}\)

\(2S=1-\frac{1}{100}\)suy ra \(2S=\frac{99}{100}\)

\(S=\frac{99}{100}:2=\frac{99}{200}\)

Đặng Phương Thảo
15 tháng 7 2015 lúc 8:04

Ta có 1/1.2.3 + 1/2.3.4 + 1/3.4.5 + ... + 1/98.99.100 

= 1/2 ( 1 / 1.2 - 1 / 2.3 + 1 / 2.3 - 1 / 3.4 + ...................+ 1 / 97.98 - 1 / 98.99 + 1 / 98.99 - 1 / 99.100) 

= 1 / 2 ( 1 / 1.2 - 1 / 99.100 ) 

= 4949 / 19800

Bùi Sỹ Bình
Xem chi tiết
Nhók Bạch Dương
8 tháng 2 2018 lúc 12:29

Ta xét: \(\frac{1}{1.2}-\frac{1}{2.3}=\frac{2}{1.2.3};\frac{1}{2.3}-\frac{1}{3.4}=\frac{2}{2.3.4};...;\frac{1}{98.99}-\frac{1}{99.100}=\frac{2}{98.99.100}\)

Tổng quát : \(\frac{1}{n\left(n+1\right)}-\frac{1}{\left(n+1\right)\left(n+2\right)}=\frac{2}{n\left(n+1\right)\left(n+2\right)}\). Do đó:

\(2S=\frac{2}{1.2.3}+\frac{2}{2.3.4}+...+\frac{2}{98.99.100}\)

\(=\left(\frac{1}{1.2}-\frac{1}{2.3}\right)+\left(\frac{1}{2.3}-\frac{1}{3.4}\right)-...-\left(\frac{1}{98.99}-\frac{1}{99.100}\right)\)

\(=\frac{1}{1.2}-\frac{1}{99.100}=\frac{4949}{9900}\)

Vậy \(S=\frac{4949}{9900}\)

dhfdfeef
18 tháng 2 2018 lúc 21:35

b, Ta có : \(\frac{1}{2}>\frac{57}{462}\)mà \(\frac{1}{6}+\frac{1}{24}+\frac{1}{60}+...+\frac{1}{9240}>0\)

nên A = \(\frac{1}{2}+\left(\frac{1}{2}+\frac{1}{6}+\frac{1}{24}+\frac{1}{60}+...+\frac{1}{9240}\right)>\frac{57}{462}+0=\frac{57}{462}\)

ngocdung nguyen
21 tháng 4 2018 lúc 8:46

Bạn sai Câu a.\(2S=\frac{4949}{9900}\)Vậy \(S=\frac{4949}{9900}:2=\frac{4949}{9900}\cdot\frac{1}{2}=\frac{4949}{19800}\)

Trần Hữu Đạt
Xem chi tiết
nguyễn minh châu
Xem chi tiết
Tran Minh Hang
Xem chi tiết
lê trang linh
Xem chi tiết
doraemon
Xem chi tiết
I am➻Minh
Xem chi tiết
Trần Tiến Phát
Xem chi tiết
fgdzg dfbsdgbdg
26 tháng 3 2018 lúc 13:09

Xét riêng cái biểu thức trong ngoặc nhé, đặt là B :
B = 1/6+1/24+1/60+...+1/9240
B = 1/1x2x3+1/2x3x4+1/3x4x5+...+1/20x21x22
B = (1/1x2 - 1/2x3) : 2 + (1/2x3-1/3x4) : 2 + (1/3x4 - 1/4x5) : 2 +...+ (1/20x21 - 1/21x22) : 2
B = (1/1x2-1/2x3+1/2x3-1/3x4+1/3x4-...-1/21x22) : 2 
B = (1/2 - 1/462) : 2 
B = 115/462
=> A = 1/2 x 115/462 > 1/2 x 114/462 = 57/462 (đpcm)

fgdzg dfbsdgbdg
26 tháng 3 2018 lúc 13:10

k minh nhe

dung 100% day nhe